WikiJournal of Science/Can each number be specified by a finite text?

From Wikiversity
Jump to navigation Jump to search

WikiJournal of Science
Open access • Publication charge free • Public peer review • Wikipedia-integrated

WikiJournal of Science is an open-access, free-to-publish, Wikipedia-integrated academic journal for science, mathematics, engineering and technology topics. WJS WikiJSci Wiki.J.Sci. WikiJSci WikiSci WikiScience Wikiscience Wikijournal of Science Wikiversity Journal of Science WikiJournal Science Wikipedia Science Wikipedia science journal STEM Science Mathematics Engineering Technology Free to publish Open access Open-access Non-profit online journal Public peer review

<meta name='citation_doi' value='10.15347/WJS/2020.008'>

Article information

Author: Boris Tsirelson[a][i] 

See author information ▼
  1. Tel Aviv University (retired), † (deceased)
  1. boris.tsirelson@gmail.com

Abstract

Contrary to popular misconception, the question in the title is far from simple. It involves sets of numbers on the first level, sets of sets of numbers on the second level, and so on, endlessly. The infinite hierarchy of the levels involved distinguishes the concept of "definable number" from such notions as "natural number", "rational number", "algebraic number", "computable number" etc.


Introduction

The question in the title may seem simple, but is able to cause controversy and trip up professional mathematicians. Here is a quote from a talk "Must there be numbers we cannot describe or define?" [1] by J.D. Hamkins.

The math tea argument
Heard at a good math tea anywhere:
“There must be real numbers we cannot describe or define, because there are uncountably many real numbers, but only countably many definitions.”
Does this argument withstand scrutiny?

See also "Maybe there's no such thing as a random sequence" [2] by P.G. Doyle (in particular, on pages 6,7 note two excerpts from A. Tarski [42]). And on Wikipedia one can also find the flawed "math tea" argument on talk pages and obsolete versions of articles.[1] [2] And elsewhere on the Internet.[3] I, the author, was myself a witness and accomplice. I shared and voiced the flawed argument in informal discussions (but not articles or lectures). Despite some awareness (but not professionalism) in mathematical logic,[4] I was a small part of the problem, and now I try to become a small part of the solution, spreading the truth.

Careless handling of the concept "number specified by a finite text" leads to paradoxes; in particular, Richard's paradox.

Richard's paradox

The paradox begins with the observation that certain expressions of natural language define real numbers unambiguously, while other expressions of natural language do not. For example, "The real number the integer part of which is 17 and the th decimal place of which is 0 if is even and 1 if is odd" defines the real number 17.1010101... = 1693/99, while the phrase "the capital of England" does not define a real number.

Thus there is an infinite list of English phrases (such that each phrase is of finite length, but lengths vary in the list) that define real numbers unambiguously. We first arrange this list of phrases by increasing length, then order all phrases of equal length lexicographically (in dictionary order), so that the ordering is canonical. This yields an infinite list of the corresponding real numbers: Now define a new real number as follows. The integer part of is 0, the th decimal place of is 1 if the th decimal place of is not 1, and the th decimal place of is 2 if the th decimal place of is 1.

The preceding two paragraphs are an expression in English that unambiguously defines a real number Thus must be one of the numbers However, was constructed so that it cannot equal any of the This is the paradoxical contradiction.

(Quoted from Wikipedia.)


See also Definability paradoxes by Timothy Gowers.

In order to ask (and hopefully solve) a well-posed question we have to formalize the concept "number specified by a finite text" via a well-defined mathematical notion "definable number". What exactly is meant by "text"? And what exactly is meant by "number specified by text"? Does "specified" mean "defined"? Can we define such notions as "definition" and "definable"? Striving to understand definitions in general, let us start with some examples.

136 notable constants are collected, defined and discussed in the book "Mathematical constants" by Steven Finch [3]. The first member of this collection is "Pythagoras’ Constant, "; the second is "The Golden Mean, "; the third "The Natural Logarithmic Base, e"; the fourth "Archimedes’ Constant, "; and the last (eleventh) in Chapter 1 "Well-Known Constants" is "Chaitin’s Constant".

Each constant has several equivalent definitions. Below we take for each constant the first (main) definition from the mentioned book.

  • The first constant is defined as the positive real number whose product by itself is equal to 2. That is, the real number satisfying and
  • The second constant is defined as the real number satisfying and
  • The third constant is defined as the limit of as That is, the real number satisfying the following condition:
for every there exists such that for every satisfying and holds
The same condition in symbols:
logical notation

"and"      "or"      "implies"      "not"      "for every"      "there exists (at least one)"      "there exists one and only one"          (a longer list).


We note that these three definitions are of the form "the real number satisfying " where is a statement that may be true or false depending on the value of its variable ; in other words, not a statement when is just a variable, but a statement whenever a real number is substituted for the variable. Such is called a property of , or a predicate (on real numbers).

Not all predicates may be used this way. For example, we cannot say "the real number satisfying " (why "the"? two numbers satisfy, one positive, one negative), nor "the real number satisfying " (no such numbers). In order to say "the real number satisfying " we have to prove existence and uniqueness:

existence: (in words: there exists such that );
uniqueness: (in words: whenever and satisfy they are equal).

In this case one says "there is one and only one such x" and writes "".

The road to definable numbers passes through definable predicates. We postpone this matter to the next section and return to examples.

  • The fourth constant is defined as the area enclosed by a circle of radius 1.

This definition involves geometry. True, a lot of equivalent definitions in terms of numbers are well-known; in particular, according to the mentioned book, this area is equal to However, in general, every branch of mathematics may be involved in a definition of a number; existence of an equivalent definition in terms of (only) numbers is not guaranteed.

The last example is Chaitin's constant. In contrast to the four constants (mentioned above) of evident theoretical and practical importance, Chaitin's constant is rather of theoretical interest. Its definition is intricate. Here is a simplified version, sufficient for our purpose.[5]

  • The last constant is defined as the sum of the series where is equal to 1 if there exist natural numbers such that otherwise and is a polynomial in 10 variables, with integer coefficients, such that the sequence is uncomputable.

Hilbert’s tenth problem asked for a general algorithm that could ascertain whether the Diophantine equation has positive integer solutions given arbitrary polynomial with integer coefficients. It appears that no such algorithm can exist even for a single and arbitrary when is complicated enough. See Wikipedia: computability theory, Matiyasevich's theorem; and Scholarpedia:Matiyasevich theorem.

The five numbers are defined, thus, should be definable according to any reasonable approach to definability. The first four numbers are computable (both theoretically and practically; in fact, trillions, that is, millions of millions, of decimal digits of are already computed), but the last number is uncomputable. How so? Striving to better understand this strange situation we may introduce approximations to the numbers as follows: is equal to 1 if there exist natural numbers less than such that otherwise here is arbitrary. For each we have as that is, the sequence is increasing, and converges to Also, this sequence is computable (given just check all the points ). Now we introduce approximations to the number as follows: We have (as ), and the sequence is computable. A wonder: a computable increasing sequence of rational numbers converges to a uncomputable number!

For every there exists such that such depending on denote it and get moreover, for all In order to compute up to it suffices to compute Doesn't it mean that is computable? No, it does not, unless the sequence is computable. Well, these numbers need not be optimal, just large enough. Isn't large enough? Amazingly, no, this is not large enough. Moreover, is not enough. And even the "power tower" is still not enough!

Here is the first paragraph from a prize-winning article by Bjorn Poonen [4]:

Does the equation have a solution in integers? Yes: (3, 1, 1), for instance. How about Again yes, although this was not known until 1999: the smallest solution is (−283059965, −2218888517, 2220422932). And how about This is an unsolved problem.

Given that the simple Diophantine equation has solutions for but only beyond we may guess that the "worst case" Diophantine equation needs very large In fact, the sequence has to be uncomputable (otherwise would be computable, but it is not). Some computable sequences grow fantastically fast. See Wikipedia: "Ackermann function", "Fast-growing hierarchy". And nevertheless, no one of them bounds from above the sequence Reality beyond imagination!

Every computable number is definable, but a definable number need not be computable. Computability being another story, we return to definability.

From predicates to relations

Recall the five definitions mentioned in the introduction. They should be special cases of a general notion "definition". In order to formalize this idea we have to be more pedantic than in the introduction. "Nothing but the hard technical story is any real good" (Littlewood, A Mathematician's Miscellany, page 70); exercises are waiting for you.

All mathematical objects (real numbers, limits, sets etc.) are treated in the framework of the mainstream mathematics, unless stated otherwise. Alternative approaches are sometimes mentioned in Sections 9, 10. Naive set theory suffices for Sections 27; axiomatic set theory is used in Sections 810.

A definition is a text in a language. A straightforward formalization of such notions as "definition" and "definable" uses "formal language" (a formalization of "language") and other notions of model theory. Surprisingly, there is a shorter way. Operations on sets are used instead of logical symbols, and relations instead of predicates.

"However, predicates have many different uses and interpretations in mathematics and logic, and their precise definition, meaning and use will vary from theory to theory." (Quoted from Wikipedia.) Here we use predicates for informal explanations only; on the formal level they will be avoided (replaced with relations).

The number was defined as the real number such that where is the predicate " and ". This predicate is the conjunction of two predicates and the first being "", the second "". The single-element set corresponding to the predicate is the intersection of the sets and (Here and everywhere, is the set of all real numbers.)

set notation

  " is the set of all such that "        " belongs to "        union        intersection      set difference        Cartesian product        real line        Cartesian plane          and more, more, more.


This is instructive. In order to formalize a definition of a number via its defining property, we have to deal with sets of numbers, and more generally, relations between numbers.

Also, is the product and is the sum But what is "product", "sum", "" and ""? The answer is given by the axiomatic approach to real numbers: they are a complete totally ordered field. It means that addition, multiplication and order are defined and have the appropriate properties. Thus, 0 is defined as the real number satisfying the condition Similarly, is defined as the real number satisfying the condition

Now we need predicates with two and more variables. The order is a binary (that is, with two variables) predicate "". Addition is a ternary (that is, with three variables) predicate "". Similarly, multiplication is a ternary predicate "" (denoted also "" or "").

Each unary (that is, with one variable) predicate on real numbers leads to a set of real numbers, a subset of the real line Likewise, each binary predicate on reals leads to a set of pairs of real numbers, a subset of the Cartesian plane the latter being the Cartesian product of the real line by itself. On the other hand, a binary relation on is defined as an arbitrary subset of

Thus, each binary predicate on reals leads to a binary relation on reals. If we swap the variables, that is, turn to another predicate that is then we get another relation inverse (in other words, converse, or opposite) to the former relation (generally different, but sometimes the same).

Similarly, each ternary predicate on reals leads to a ternary relation on reals; and, changing the order of variables, we get ternary relations (generally, different) corresponding to 6 permutations of 3 variables. And generally, each n-ary predicate on reals leads to a n-ary relation on reals (a subset of ); and, changing the order of variables, we get such relations. The case is included (for unification); a unary relation on reals (called also property of reals) is a subset of

Thus, on reals, the order is the binary relation the addition is the ternary relation and the multiplication is the ternary relation Still, we cannot forget predicates until we understand how to construct new relations out of these basic relations. For example, how to construct the binary relation and the unary relation We know that if a predicate is the conjunction of two predicates, then it leads to the intersection of the corresponding sets. Similarly, the disjunction leads to the union , and the negation leads to the complement Also, the implication leads to and the equivalence leads to The same holds for n-ary predicates; the disjunction still corresponds to the union the negation to the complement etc. But what to do when is or is or is etc?

This question was answered, in context of axiomatic set theory, in the first half of the 20th century.[6] A somewhat different answer, in the context of definability, was given by van den Dries in 1998 [5], [6] and slightly modified by Auke Bart Booij in 2013 [7]; see also Macintyre 2016 [8, "Defining First-Order Definability"]. Here is the answer (slightly modified).

First, in addition to the Boolean operations (union and complement; intersection is superfluous, since it is complement of the union of complements) on subsets of we introduce permutation of coordinates; for example (), and in general,

where is an arbitrary permutation of

In particular, permutation of coordinates in a binary relation gives the inverse relation. For example, the inverse to is And, by the way, the intersection of these two is the relation (corresponding to the predicate "").

Second, set multiplication, in other words, Cartesian product by : that is,

turns a -ary relation to a relation that is formally -ary, but the last variable is unrelated to others.

Now, returning to a predicate of the form we treat the corresponding ternary relation as the intersection of two ternary relations and and as the Cartesian product of the binary relation by being inverse to the relation (corresponding to the given predicate ); and as obtained (by permutation of coordinates) from the Cartesian product (by ) of the relation corresponding to the given predicate

Third, the projection; for example (), and in general,

it turns a -ary relation to a -ary relation. For the set is also called the domain of the binary relation

Now, returning to a predicate of the form we rewrite it as "" and treat the corresponding binary relation as the projection of the ternary relation and as a permutation of the Cartesian product

What if is ""? Then we rewrite it as "" and get the complement of the projection of the complement of the relation corresponding to

So, we accept the 3 given relations (order, addition, multiplication) as "definable", and we accept the 5 operations (complement, union, permutation, set multiplication, projection) for producing definable relations out of other definable relations. Thus we get infinitely many definable relations (unary, binary, ternary and so on).

More formally, these relations are called "first-order definable (without parameters) over "; but, less formally, "definable over" is often replaced with "definable in" (and sometimes "definable from"); "without parameters" is omitted throughout this essay; also "first order" and "over " are often omitted in this section. See Wikipedia: Definable set: Definition; The field of real numbers.

Generally, starting from a set (not necessarily the real line) and some chosen relations on this set (including the equality relation if needed), and applying the 5 operations (complement, union, permutation, set multiplication, projection) repeatedly (in all possible combinations), one obtains an infinite collection of relations (unary, binary, ternary and so on) on the given set. Every such collection of relations is called a structure (Booij [7]), or a VDD-structure (Brian Tyrrell [9]) on the given set. According to Tyrrell [9, page 3], "The advantage of this definition is that no model theory is then needed to develop the theory". The technical term "VDD structure" (rather than just "structure" used by van den Dries and Booij) is chosen by Tyrrell "to prevent a notation clash" (Tyrrell [9, page 2]), since many other structures of different kinds are widely used in mathematics. "VDD" apparently refers to van den Dries who pioneered this approach. But let us take a shorter term "D-structure", where "D" refers to "definable" and "Dries" as well. The D-structure obtained (by the 5 operations) from the chosen relations, in other words, generated by these relations, is the smallest D-structure containing these relations.

Generality aside, we return to the special case, the D-structure of definable relations on the real line defined above (generated by order, addition and multiplication; though, the order appears to be superfluous).

  • Exercise 2.1. Prove that a relation is definable in if and only if it is definable in  Hint: if and only if

We say that a number is definable, if the single-element set is a definable unary relation.

  • Exercise 2.2. Prove that the numbers 0 and 1 are definable.  Hint: recall "" and "".
  • Exercise 2.3. Prove that the sum of two definable numbers is definable.  Hint:
  • Exercise 2.4. Prove that the number is definable.  Hint:
  • Exercise 2.5. Prove that the number is definable.  Hint:
  • Exercise 2.6. Prove that the golden ratio is definable.
  • Exercise 2.7. Prove that the binary relation "" is definable.  Hint:

In contrast, the ternary relation "" is not definable. Moreover, the binary relation is not definable. The problem is that all relations definable in are semialgebraic sets over (the subring of) integers.

Proofs

Theorem. All relations definable in are semialgebraic sets over integers.

Proof. The two relations "", "" are semialgebraic (evidently). Two operations, permutation and set multiplication, applied to semialgebraic relations, give semialgebraic relations (evidently). The third, projection operation, applied to a semialgebraic relation, gives a semialgebraic relation by the Tarski–Seidenberg theorem [10, Theorem 2.76].

Theorem. If and then the binary relation is not semialgebraic.

Proof. Assume the contrary. Then the function on being semialgebraic, must be algebraic [10, Prop. 2.86], [11, Corollary 3.5]). It means existence of a polynomial (not identically 0) such that for all It follows that for all complex numbers Taking we get for all integer Therefore for all complex (otherwise the polynomial cannot have infinitely many roots). Similarly, taking we get for all complex and all therefore everywhere; a contradiction.


Thus, we cannot define the number via in this framework. Also, only algebraic numbers are definable in this framework.

Each natural number is definable, which does not mean that the set of all natural numbers is definable (in ). In fact, it is not!

Proof

Follows immediately from the lemma below.

Lemma. For every semialgebraic subset of there exists such that either or

Proof. First, the claim holds for every set of the form where is a polynomial, since either as or as or is constant. Second, a Boolean operation (union, complement), applied to sets that satisfy the claim, gives a set that satisfies the claim (evidently).


We could accept the set of natural numbers as definable, that is, turn to definability in but does it help to define the number ? Surprisingly, it does! "[...] then the situation changes drastically" (van den Dries [5, Example 1.3]). See also Booij [7, page 17]: "[...] if we add the seemingly innocent set Z to the tame structure of semialgebraic sets, we get a wild structure [...]"

Beyond the algebraic

In this section, "definable" means "first order definable in ". In other words, the real line is endowed with the D-structure generated by addition, multiplication, and the set of natural numbers. Good news: we'll see that the five numbers discussed in Introduction, are definable. Bad news: in addition to their usual definitions we'll use Diophantine equations, computability and Matiyasevich's theorem (mentioned in Introduction in relation to Chaitin's constant). The reader not acquainted with computability theory should rely on intuitive idea of computation (instead of formal proofs of computability), and consult the linked Wikipedia article for computability-related notions ("recursively enumerable", "computable sequence"). Alternatively, the reader may skip to Section 5; there, usual definitions will apply, no computability needed.

Every Diophantine set

(where is a polynomial with integer coefficients), treated as a subset of is a definable n-ary relation. And every recursively enumerable set is Diophantine.

For every computable sequence of natural numbers, the binary relation is recursively enumerable, therefore definable.

In particular, the binary relation " and " is definable, as well as " and ". Now (at last!) the number is definable, via more formally, is the real number satisfying the condition

This is not quite the definition mentioned in Introduction, but equivalent to it.

Similarly, for every convergent computable sequence of rational numbers, its limit is a definable number. In other words, every limit computable real number is definable.

Every computable real number is limit computable, therefore definable. In particular, the number is computable, therefore definable.

Chaitin's constant is not computable, but still, limit computable (recall Introduction: it is the limit of a computable increasing sequence of rational numbers), therefore definable. So, all the five constants discussed in Introduction (taken from the book "Mathematical constants") are definable. Moreover, all the constants discussed in that book are definable.

On the other hand, if we choose a number between 0 and 1 at random, according to the uniform distribution, we almost surely get an undefinable number, because the definable numbers are a countable set.

Of course, such a randomly chosen undefinable number is not an explicit example of undefinable number. It may seem that "explicit example of undefinable number" is a patent nonsense, just as "defined undefinable number". But no, not quite nonsense, see Section 4.

An infinite sequence of real numbers is nothing but the binary relation if this binary relation is definable, we say that the sequence is definable. If a sequence is definable, then all its members are definable numbers. However, a sequence of definable numbers is generally not definable.

  • Exercise 3.1. If a definable sequence converges, then its limit is a definable number. Prove it.

A function is nothing but the binary relation "", that is, if this binary relation is definable, we say that the function is definable. An arbitrary binary relation is a function if and only if for every there exists one and only one such that

  • Exercise 3.2. If is a definable function and is a definable number, then is a definable number. Prove it.

However, a function that has definable values at all definable arguments is generally not definable.

  • Exercise 3.3. If a definable function is differentiable, then its derivative is a definable function. Prove it.  Hint: the derivative is the limit of...
  • Exercise 3.4. If a definable function is continuous, then its antiderivative is definable if and only if is a definable number. Prove it.  Hint:

Similarly to the number we can treat the exponential function First, the relation is definable (since is a computable function of ). Second, the relation is definable, since is the limit of as tends to infinity and tends to more formally (but still not completely formally...), if and only if

here is the set of integers (evidently definable).

The cosine function may be treated via complex numbers and Euler's formula First, the real part of the complex number is a computable function of Second, its limit as tends to infinity and tends to is equal to the real part of the complex number

Note that the exponential integral and the sine integral are definable nonelementary functions.

Definable functions can be pathological and disrespect dimension. In particular, there is a definable one-to-one correspondence between the (two-dimensional) square and a subset of the (one-dimensional) interval which will be used in Section 6. Here is a way to this fact.

Given two numbers we consider their decimal digits: where for each and the set is infinite (since we represent, say, as rather than ); and similarly We interweave their digits, getting a third number The ternary relation between such is a function Not all numbers of are of the form (for example, is not), which does not matter. It does matter that are uniquely determined by that is, implies In other words, is an injection

Denoting by the -th decimal digit of we have here is the integer part of and is the fractional part of

  • Exercise 3.5. The integer part function is definable. Prove it.  Hint:
  • Exercise 3.6. The function is definable. (See Booij [7, Lemma 3.4].) Prove it.  Hint:
  • Exercise 3.7. The function is definable. Prove it.  Hint:
  • Exercise 3.8. Generalize the previous exercise to  Hint: consider

Explicit example of undefinable number

We construct such example in two steps. First, we enumerate all numbers definable in ("first order" is meant but omitted, as before); that is, we construct a sequence of real numbers that contains all numbers definable in (and only such numbers). Second, we construct a real number not contained in this sequence.

The second step is well-known and simple, so let us do it now, for an arbitrary sequence of real numbers. We construct a real number via its decimal digits, as and we choose each to be different from the n-th digit (after the decimal point) of the absolute value of To be specific, let us take if for some integer and otherwise. Then since the integral part of being of the form for integer is different from the integral part of the latter being of the form for integer and (either or ). This is an instance of Cantor's diagonal argument.

Now we start constructing a sequence of real numbers that contains all numbers definable in (and only such numbers). These numbers being elements of single-element subsets of definable in and these subsets being unary relations, we enumerate all relations (unary, binary, ...) definable in These are obtained from the three given relations (addition, multiplication, "naturality") via the 5 operations (complement, union, permutation, set multiplication, projection) applied repeatedly. We may save on permutations by restricting ourselves to adjacent transpositions, that is, permutations that swap two adjacent numbers and leave intact other numbers of this is sufficient, since every permutation is a product of some adjacent transpositions. We start with the three given relations

"addition"
"multiplication"
"naturality"

and apply to them the five operations (whenever possible). The first operation "complement" gives

The "union" operation gives

The "permutation" operation (reduced to adjacent transpositions), applied to the ternary relation gives two relations namely, (equal to due to commutativity, but we do not bother) and we apply the same to getting Further, "set multiplication" gives the 4-ary relation

similarly and . The most remarkable "projection" operation gives

(in fact, ), and similarly

The first 16 relations are thus constructed. On the next iteration we apply the 5 operations to these 16 relations (whenever possible; though, some are superfluous) and get a longer finite list. And so on, endlessly. A bit cumbersome, but really, a routine exercise in programming, isn't it? Well, it is, provided however that the "programming language" stipulates the data type "relation over " and the relevant operations on relations. By the way, equality test for relations is not needed (unless we want to skip repetitions); but test of existence and uniqueness (for unary relations), and extraction of the unique element, are needed for the next step.

Now we are in position to construct for each we check, whether the relation is of the form for or not; if it is, we take otherwise (Note that whenever the relation is not unary.)

Applying the diagonal argument (above) to this sequence we construct a real number not contained in the sequence, therefore, not definable in

This number is defined, but not in Why not? Because the definition of involves a sequence of relations on Sequences of numbers are used in Section 3, but sequences of relations are something new, beyond the first order. (See Wikipedia: First-order logic, Second-order logic.)

Is there a better approach? Could we define in the same sequence or maybe another sequence containing all computable numbers, by a clever trick? No, this is impossible. For every sequence definable in the diagonal argument gives a number definable in and not contained in the given sequence.

Second order

We introduce second-order definability in The set of natural numbers is second-order definable in as we'll see soon. In contrast to the first order definability, usual definitions of mathematical constants will apply without recourse to computability and Diophantine sets.

A second-order predicate is a predicate that takes a first-order predicate as an argument. Likewise, a second-order relation is a relation between relations. For example, the binary relation between a function and its derivative may be thought of as a relation between two binary relations: first, the relation between real numbers and second, the relation between real numbers First order definability of a real number involves definable first-order relations (between real numbers). Second order definability of a real number involves definable second-order relations (between first-order relations). Here is a possible formalization of this idea.

We introduce the set

that contains, on one hand, all tuples (finite sequences) of real numbers (for all ; here we do not distinguish 1-tuples from real numbers), and on the other hand, all -ary relations on (for all ). Here is the set of all subsets (that is, the power set) of the real line in other words, of unary relations on is the set of all subsets (that is, the power set) of the Cartesian plane in other words, of binary relations on and so on. On this set we introduce two relations:

  • membership, the binary relation it says that the given -tuple belongs to the given -ary relation;
  • "appendment", the ternary relation it says that the latter tuple results from the former tuple by appending the given real number.

The two ternary relations on addition and multiplication, may be thought of as ternary relations on (since ):

  • addition:
  • multiplication:

We endow with the D-structure generated by the four relations (membership, appendment, addition, multiplication). All relations on that belong to this D-structure will be called second-order definable. In the rest of this section, "definable" means "second-order definable", unless stated otherwise.

  • Exercise 5.1. The set of all tuples and the set of all relations are definable subsets of Prove it.  Hint: first, the set of all tuples is where is the appendment relation; second, take the complement.
  • Exercise 5.2. The set of all real numbers is a definable subset of Prove it.  Hint: where is the appendment relation.
  • Exercise 5.3. Each is a definable subset of Prove it.  Hint: where is the appendment relation.
  • Exercise 5.4. The set of all sets of real numbers (that is, unary relations) is a definable subset of Prove it.  Hint: for we have apply the projection to
  • Exercise 5.5. For each the set of all -ary relations is a definable subset of Prove it.  Hint: similar to the previous exercise.
  • Exercise 5.6. If is a definable set of subsets of , then the union of all these subsets is a definable set (of real numbers). Prove it.  Hint: for we have take the projection of
  • Exercise 5.7. Do the same for the intersection  Hint: consider (But what if is empty?)
  • Exercise 5.8. Generalize the two exercises above to and so on.  Hint: now is a tuple.

In particular, taking a single-element set we see that definability of implies definability of The converse holds as well (see below).

  • Exercise 5.9. If a set (of real numbers) is definable, then the set (of all subsets of ) is definable. Prove it.  Hint: for we have consider
  • Exercise 5.10. Do the same for the set (of all supersets of ).  Hint: similarly to the previous exercise, consider
  • Exercise 5.11. Generalize the two exercises above to and so on.

Remark.   These 11 exercises (above) do not use addition and multiplication, nor any properties of real numbers. They generalize readily to a more general situation. One may start with an arbitrary set (rather than the real line ), consider the set constructed from as above (all tuples and all relations), endow by a D-structure such that the two relations on , membership and appendment, are definable, and generalize the 11 exercises to this case.

Taking the intersection of the set of subsets and the set of supersets we see that definability of implies definability of the single-element set (called singleton) So, is definable if and only if is definable. And still (by convention, as before) a real number is definable if and only if is definable.

Does it mean that, for example, numbers are definable (as well as every rational number and every algebraic number)? We know that they are first-order definable in does it follow that they are (second-order) definable in

The answer is affirmative, but needs a proof. Here we face another general question. Let be a set and its subset. Every -ary relation on is also a -ary relation on (since ). Thus, given some relations on we get two D-structures; first, the D-structure on generated by the given relations, and second, the D-structure on generated by the same relations.

Lemma.   Assume that is a definable subset of (according to the second D-structure). Then every relation on definable according to the first D-structure is also definable according to the second D-structure.

Proof

Denote the first D-structure by and the second by We know that It follows (via set multiplication) that and so on; by induction, for all Thus (via permutation),

In order to prove that we compare the five operations on relations (complement, union, permutation, set multiplication, projection) over (call them -operations) and over (-operations). We have to check that each -operation applied to relations on that belong to gives again a relation (on ) that belongs to

For the union we have nothing to check, since the -union of two relations is equal to their -union. Similarly, we have nothing to check for permutation and projection. Only set multiplication and complement need some attention.

Set multiplication. The -multiplication applied to gives We have since and

It follows (by induction) that for all

Complement. The -complement applied to gives We note that the -complement belongs to (since ), thus (since ).


Now we are in position to prove definability of the set of natural numbers. It is sufficient to prove definability of the set of all sets satisfying the two conditions and (since the intersection of all these is ). The complement is the projection of the intersection of two sets, and The former results from the (permuted) membership relation; the latter is the projection of the projection of this set being the intersection of three sets: first, second, times the addition relation; third, It follows that is definable, whence is definable.

This is instructive. In order to formalize a definition of a set via its defining property, we have to deal with sets of sets, and more generally, relations between sets.

Using again the lemma above we see that all real numbers first-order definable in are second-order definable. Section 3 gives many examples, including the five numbers discussed in Introduction. But second-order proofs of their definability are much more easy and natural.

The binary relation "" is the sequence of factorials, that is, the set It is definable, similarly to since it is the least subset of such that and Alternatively, it is definable since it is the only subset of with the following three properties: :

That is, the factorial is the only function satisfying the recurrence relation and the initial condition

  • Exercise 5.12. Partial sums of the series are a definable sequence. Prove it.
  • Exercise 5.13. The number is definable. Deduce it from the previous exercise.

In the first-order framework it is possible to treat many functions (for instance, the exponential function the sine and cosine functions the exponential integral and the sine integral ) and many relations between functions (for instance, derivative and antiderivative); arguments and values of these functions are arbitrary real numbers (not necessarily definable), but the functions are definable. Such notions as arbitrary functions (not necessarily definable), continuous functions (and their antiderivatives), differentiable functions (and their derivatives) need the second-order framework.

As was noted there, a function is nothing but the binary relation "", that is, An arbitrary binary relation is such a function if and only if for every there exists one and only one such that (existence and uniqueness). For functions defined on arbitrary subsets of the real line the condition is weaker: for every there exists at most one such that (uniqueness).

  • Exercise 5.14. (a) All satisfying the uniqueness condition are a definable subset of (b) the same holds for the existence and uniqueness condition. Prove it.
  • Exercise 5.15. All continuous functions are a definable subset of Prove it.
  • Exercise 5.16. All differentiable functions are a definable subset of Prove it.
  • Exercise 5.17. The binary relation "" is definable. That is, the set of all pairs of functions such that is a definable subset of (in other words, definable binary relation on ). Prove it.

Antiderivative can now be treated in full generality. In contrast, in the first-order framework it was treated via Riemann integral for continuous definable only. In particular, now the exponential function may be treated via where for accordingly, the constant may be treated via Alternatively, the exponential function may be treated via the differential equation (and initial condition ). Trigonometric functions may be treated via the differential equation accordingly, the constant may be treated as the least positive number such that Or, alternatively, as (via antiderivative).

This is instructive. In the second-order framework we may define functions (and infinite sequences) via their properties, irrespective of computability, Diophantine equations and other tricks of the first-order framework.

Nice; but what about second-order definable real numbers? Are they all first-order definable, or not? Even if obtained from complicated differential equations, they are computable, therefore, first-order definable in . Probably, our only chance to find a second-order definable but first-order undefinable number is, to prove that the explicit example of (first-order) undefinable number, given in Section 4, is second-order definable; and our only chance to prove this conjecture is, to formalize that section within the second-order framework.

First-order undefinable but second-order definable

Recall the infinite sequence of relations treated in Section 4. Is it second-order definable? Each belongs to the set (from Section 5); their infinite sequence is a binary relation between and (namely, the set of pairs ), thus, a special case of a binary relation on the question is, whether this relation is definable, or not. Like the sequence of factorials, it is defined by recursion. But factorials, being numbers, are first-order objects, which is why their sequence is second-order definable via its properties. In contrast, relations are second-order objects! Does it mean that third order is needed for defining their sequence by recursion?

True, the sequence of factorials is first-order definable (in ) due to its computability, via Matiyasevich's theorem. Could something like that be invented for second-order objects? Probably not.

Yet, these obstacles are surmountable. The sequence of relations may be replaced with a single relation by a kind of currying (or rather, uncurrying); the disjoint union may be used instead of the set of pairs Further, relations of different arities may be replaced with unary relations (subsets of the real line), since two real numbers may be encoded into a single real number via an appropriate definable injection and the same applies to three and more numbers (moreover, to infinitely many numbers, see Booij [7, Section 3.2]). In addition, tuples may be replaced (whenever needed) by finite sequences which provides a richer assortment of definable relations.

The distinction between tuples and finite sequences is a technical subtlety that may be ignored in many contexts, but sometimes requires attention. It is tempting to say that an ordered pair a 2-tuple (that is, tuple of length 2), and a 2-sequence (that is, finite sequence of length 2) are just all the same. However, the 2-sequence is, by definition, a function on thus, the set of two ordered pairs Surely we cannot define an ordered pair to be a set of two other ordered pairs! If sequences are defined via functions, and functions are defined via pairs, then pairs must be defined before sequences, and cannot be the same as 2-sequences. See Wikipedia: sequence (formal definition), tuples (as nested ordered pairs), and ordered pair: Kuratowski's definition. For convenience we'll denote a finite sequence by it is similar to, but different from, the tuple

We'll construct again, this time in the second-order framework, the sequence of real numbers that contains all numbers first-order definable in exactly the same sequence as in Section 4. To this end we'll construct first the disjoint union of unary relations on similar to, but different from, relations (unary, binary, ...) constructed there (that exhaust all relations first-order definable in ).

Before the unary relations we construct 4-tuples of integers (call them "instructions") imitating a program for a machine that computes Similarly to a machine language instruction, each contains an operation code, address of the first operand, a parameter or address of the second operand (if applicable, otherwise 0), and in addition, the arity of

Recall Section 4. Three relations of arities 3,3,1 are given, and lead to the next 13 relations In particular, is the complement of Accordingly, we let here, operation code 1 means "complement...", operand address 1 means "...of ", the third number 0 is dummy, and the last number 3 means that the relation is ternary. Similarly, and

Further, being the union of and we let operation code 2 means "union...", first operand address 1 means "...of ", second operand address 2 means "...and ", and again, 3 is the arity of

Further, being a permutation of we let operation code 3 means "permutation...", operand address 1 means "...of ", the parameter 1 means "swap 1 and 2", and 3 is the arity of Similarly, (in swap 2 and 3), (in swap 1 and 2), (in swap 2 and 3).

Further, being we let operation code 4 means "set multiplication", 1 refers to the operand and 4 is the arity of Similarly, and

Further, being the projection of we let operation code 5 means "projection...", 1 means "...of ", and 2 means "...is binary". Similarly,

This way, the finite sequence of natural numbers (interpreted as arities) leads to the finite sequence of 4-tuples (interpreted as instructions). Similarly, every finite sequence of natural numbers leads to the corresponding finite sequence of 4-tuples. The relation between these two finite sequences is definable; the proof is rather cumbersome, like a routine exercise in programming, but doable. Having this relation, we define an infinite sequence of 4-tuples (interpreted as the infinite "program") together with an infinite sequence by the following defining properties:

  • for every the finite sequence of 4-tuples corresponds (according to the definable relation treated above) to the finite sequence of the natural numbers that are the last (fourth) elements of the 4-tuples

In particular, the third property for states that corresponds to And for it states that corresponds to And so on.

The infinite program is ready. It could compute all relations if executed by a machine able to process relations of all arities. Is such machine available in our framework? The disjoint union could be used instead of the set of pairs but is not contained in (say) True, in practice 100-ary relations do not occur in definitions; but we investigate definability in principle (rather than in practice). We encode all relation into unary relations as follows.

We recall the definable injective functions and treated in the end of Section 3. The same works for any But we need to serve all dimensions by a single definable function. To this end we turn from sets of -tuples to sets, denote them of -sequences

  • Exercise 6.1. The set of all finite sequences of real numbers, is definable, and the binary relation "length", on is a definable function on that set. Prove it.  Hint: start with the binary relation.
  • Exercise 6.2. The function ("evaluation") is a definable real-valued function on the set Prove it.  Hint: for and we have use the two given relations on membership and appendment (consider in the definition of appendment).

We choose a definable bijection for example, and define a function by for all and

  • Exercise 6.3. The function is definable. Prove it.  Hint: for all and we have that is, for all and holds

At last, we are in position to "execute the infinite program" that is, to prove (second-order) definability of the set the disjoint union of unary relations on that encode (according to ) the relations (that exhaust all relations first-order definable in ).

We extract decode the ternary relation and require it to be (like ) the addition relation That is, we require

This condition fails to uniquely determine the set since the image of under is not the whole (not even the whole ). We prevent irrelevant points by requiring in addition that We do not repeat such reservation below.

Similarly, must encode the multiplication relation, and must encode the set of natural numbers:

These first three requirements (above) are special. Other requirements should be formulated in general, like this: for every if the first element of (the operation code) equals 1, then (...), otherwise (...). But let us consider several examples before the general case.

According to the instruction the set must encode the complement of the set encoded by :

similarly,

According to the instruction the set must encode the union of the sets encoded by and :

According to the instruction the set must encode the permutation of the set encoded by :

similarly,

According to the instruction the set must encode the Cartesian product (by ) of the set encoded by :

similarly,

According to the instruction the set must encode the projection of the set encoded by :

similarly,

Toward the general formulation. We observe that the first two cases (complement and union) are unproblematic, while the other three cases (permutation, set multiplication, and projection) need some additional effort. The informal quantifiers like "" should be replaced with "", and the needed relations between finite sequences should be generalized (and formalized).

  • Exercise 6.4. The binary relation of truncation is definable. Prove it.  Hint: use the evaluation function.
  • Exercise 6.5. The ternary relation of appendment is definable. Prove it.

Now the reader should be able to compose himself the general formulation. Also the additional condition that prevents irrelevant points should be stipulated. We conclude that the set is definable. For each we check, whether the relation encoded by is of the form for or not; if it is, we take otherwise We get the definable sequence that contains all numbers first-order definable in The next step (explained in Section 4), readily fomalized (via the function from Section 3), provides a definable number not contained in this sequence.

Fast-growing sequences

Looking at decimal digits of two real numbers, for example,

can you see, which one is "more definable"? Probably not. (Answer: is algebraic, therefore first-order definable in while is not.) Surprisingly, a kind of visualization of definability is possible in an interesting special case. The number

is transcendental (that is, not algebraic). Moreover, every number of the form with is transcendental, which follows from Roth's theorem.

  • Exercise 7.1. If is a definable sequence of natural numbers, strictly increasing (that is, ), then the number is definable. Prove it both in the framework of Section 3 (first-order definability in ) and the framework of Section 5 (second-order definability).  Hint: and
  • Exercise 7.2. If a number with is definable, then the sequence is definable. Prove it in the framework of Section 5 (second-order definability).  Hint: for every is the least such that

Note that the sequence is defined by its property, which works only in the second-order framework. The first-order framework requires an explicit relation between and Nevertheless, the claim of Exercise 7.2 holds also in the framework of Section 3.

Remark about a proof

It is easy to obtain the sequence out of the sequence of sums of the digits The problem is that in the first-order framework we cannot define just by the property "". Yet, this obstacle is surmountable; we can computably encode by natural numbers all tuples of natural numbers. (A similar trick was used in Section 6.)


Thus, in order to get a first-order undefinable but second-order definable real number, it is sufficient to find a first-order undefinable but second-order definable strictly increasing sequence of natural numbers. This can be made similarly to Sections 4, 6, replacing Cantor's diagonal argument with the following fact:

  • For every sequence of sequences (of numbers) there exists a strictly increasing sequence (of numbers) that overtakes all the given sequences (of numbers).

The proof is immediate: take where the number is the -th element of the -th given sequence; then clearly whenever

  • Exercise 7.3. If the ternary relation is definable, then the binary relation is definable. Prove it.  Hint:

Reusing the construction of Section 6, we enumerate all sequences of natural numbers, definable in the framework of Section 3, by enumeration definable in the framework of Section 5, and then overtake them all by a strictly increasing sequence of natural numbers, definable in the framework of Section 5.

To fully appreciate the incredible growth rate of this sequence, we note that it overtakes all computable sequences, as well as an extremely fast-growing sequence mentioned in Introduction. Recall and discussed there. In the framework of Section 3, the ternary relation being recursively enumerable (therefore Diophantine) is definable; and the binary relation is definable, since Defining as the least such that we observe that the sequence is definable (since ). On the other hand, as noted in Introduction, this sequence cannot be bounded from above by a computable sequence.

More discussions of large numbers are available, see Scott Aaronson [7], John Baez [8] and references therein. A quote from Aaronson (pages 11-12):

You defy him to name a bigger number without invoking Turing machines or some equivalent. And as he ponders this challenge, the power of the Turing machine concept dawns on him.

Definability could be mentioned here along with Turing machines.

Definable but uncertain

Two sets are called equinumerous if there exists a one-to-one correspondence between them. In particular, two subsets of are equinumerous if (and only if) We see that the binary relation "equinumerosity" on is second-order definable.

Some subsets of are equinumerous to for some (these are finite sets). Others may be equinumerous to (these are called countable, or countably infinite), or (these are called sets of cardinality continuum), or... what else? Can a set be more than countable but less than continuum?

This seemingly innocent question is one of the most famous in set theory,[9] the first among the Hilbert's problems. The answer was expected to be "no such sets", which is the continuum hypothesis (CH); Georg Cantor tried hard to prove it, in vain; Kurt Gödel proved in 1940 that CH cannot be disproved within the axiomatic set theory called ZFC, and hoped that new axioms will disprove it;[10] Paul Cohen proved in 1963 that CH cannot be proved within ZFC, and felt intuitively that it is obviously false.[11] Nowadays some experts hope to find "the missing axiom", others argue that this is hopeless.[12]

A wonder: million published theorems[13] in all branches of mathematics formally are deduced from the 9 axioms of ZFC; they answer, affirmatively or negatively, million mathematical questions; some questions remain open, waiting for solutions in the ZFC framework; but the continuum hypothesis is an exception![14]

Back to definability. Consider the set of all subsets of that are more than countable but less than continuum. We do not know, whether is empty or not, but anyway, we know that is second-order definable. We define a number by the following property:

That is, is if CH is true, and otherwise. This is a valid definition; is second-order definable; but we cannot know, is it or Each one of the two equalities, and could be added (separately!) to the axioms of ZFC without contradiction (assuming, of course, that ZFC itself is consistent); according to the model theory, it means existence of two models of ZFC, one with the other with In this sense, is model dependent.

Is computable? Yes, it is, just because and are computable numbers, and is one of these. You might feel bothered, even outraged, but this is a valid argument. Compare it with the well-known proof that an irrational elevated to an irrational power may be rational: is either rational (which gives the needed example), or irrational, in which case gives the needed example.[15] Seeing this, some retreat to intuitionism, but almost all mathematics is classical, it accepts the law of excluded middle and cannot arbitrarily disallow it in some cases.

So, what is the algorithm for computing Surely the definition of an algorithm disallows such condition as "if CH holds, then" within an algorithm. However, it cannot disallow a model-dependent algorithm if CH holds then else where is a (trivial) algorithm that computes the number 1, and computes The conditioning "if CH holds, then" is allowed outside the algorithms (similarly, the conditioning "if is rational, then" is allowed outside the formulas). If you are unhappy with the affirmative answer to the question "is computable?", ask a different question: "is computable by a model-independent algorithm?" The answer is negative (see below).

On the other hand, definability of the number is established by a kind of "generalized algorithm" able to process second-order objects (real numbers, relations between these, and relations between relations; recall the "program" in Section 6). This "generalized algorithm" is model independent, but its output is model dependent.

In contrast, the number is model independent; for every rational number one of the two inequalities is provable in ZFC. The same applies to the numbers discussed in Introduction, since each of these numbers can be computed by a model independent algorithm. If a number is computable by a model-independent algorithm, then this number is both model independent and computable.

What about Chaitin's constant It is limit computable by a model-independent algorithm. Also, it is first-order definable (in ), and the first-order framework disallows questions (such as CH) about arbitrary sets of numbers, thus, one might hope that is model independent. But it is not!

Here we need one more fact about The sequence of its binary digits is not just uncomputable, that is, the set is not just non-recursive, but moreover, this set belongs to "the most important class of recursively enumerable sets which are not recursive",[16] the so-called creative sets, or equivalently, complete recursively enumerable sets. Basically, it means that this sequence contains answers to all questions of the form "does the natural number belong to the recursively enumerable set " And in particular(!), all questions of the form "can the statement be deduced from the theory ZFC?", since in ZFC (and many other formal theories as well) the set of (numbers of) provable statements is recursively enumerable. Taking to be the negation of something provable (for instance, ) we get the question "is ZFC consistent?" answered by one of the binary digits of whose number can be computed; if this is then ZFC is consistent; if is then ZFC is inconsistent. However, by a famous Gödel theorem, this question cannot be answered by ZFC itself! Assuming that ZFC is consistent we have but this truth is not provable (nor refutable) in ZFC. (In fact, it is provable in ZFC+large cardinal axiom.) Therefore, in some models of ZFC we have in others which shows that is model dependent. Moreover, there are versions of such that every binary digit of is model dependent [22], [23].

Yet the (first-order) case of is less bothering that the (second-order) case of since we still believe that Adding the axiom "", that is, "ZFC is inconsistent" to ZFC we get a theory that is consistent (assuming, of course, that ZFC itself is consistent) but not -consistent. This strange theory claims existence of a proof of "" in ZFC, of a finite length this length being a natural number. And nevertheless, this theory claims that and so on, endlessly. (Beware of the elusive distinction between two phrases, "for each it claims that " and "it claims that ".) Every model of this strange theory contains more natural numbers than the usual In mathematical logic we must carefully distinguish between two concepts of a natural number, one belonging to a theory, the other to its metatheory. In particular, when saying "for every rational number one of the two inequalities is provable in ZFC" we should mean that is the ratio of two metatheoretical natural numbers.

Using as binary digits an infinite sequence of independent "yes/no" parameters of models of ZFC we get a model dependent definable number whose possible values are all real numbers. More exactly, the following holds in the metatheory: for every real number there exists a model of ZFC (assuming, of course, that ZFC itself is consistent) whose natural numbers (and therefore rational numbers) are the same as in the metatheory, and for every rational number the inequality holds in the model if and only if Is this possible in the first or second order framework? I do not know. But in the third order framework this is possible, as suggested by the generalized continuum hypothesis.[17]

Higher orders; set theory

Recall the transition from first-order definability to second-order definability (Section 5); from the set of all real numbers to the set of all tuples and relations over and the D-structure on generated by the D-structure on and two relations, membership and appendment, on The next step suggests itself: the set of all tuples and relations over with the D-structure on generated by the D-structure on and two relations, membership and appendment, on formalizes third-order definability. This way we may introduce infinitely many orders of definability, or where and so on. Similarly to Section 6 we can prove that each order brings new definable real numbers (and new, faster-growing sequences of natural numbers, recall Section 7).

But this is only the tip of the iceberg. The union of all these sets, endowed with the D-structure generated by the given D-structures on all formalizes a new, transfinte order of definability, and starts a new sequence of orders. Should we denote them by ? What about ? How high is this hierarchy? Is it countable, or not?

Transfinite hierarchies are investigated by set theory (see Wikipedia:Set theory, and Section "Some ontology" there). Surprisingly, set theory does not need the field of real numbers as the starting point; not even the set of natural numbers. A wonder: set theory is able to start from nothing and get everything![18]

The cumulative hierarchy starts with the empty set, denoted by or the number defined as just another name of the empty set, and stage zero, denoted by and defined as still another name of the empty set. On the next step we consider the set of all subsets of . There is only one subset of the empty set itself, thus ; we define the number to be and stage one . Similarly, is the two-element set stage two. Somewhat dissimilarly, is the four-element set its three-element subset is (by definition) the number and is the third stage. More generally, for . Thus, is a set of elements(!), while is its subset of elements.

  • Exercise 9.1. Prove it.  Hint: .

Here we face crossroads. One way is to treat the union of all as the class of all sets (a proper class, not a set). This way leads to the finite set theory (see Takahashi [27], Baratella and Ferro [28], and others; see also Wikipedia:General set theory). The other way is to treat the union of all as an infinite set, its infinite subset as the first transfinite ordinal number, and as the first transfinite stage of the cumulative hierarchy. This way leads to the set theory widely accepted by the mainstream mathematics.[19]

Finite set theory

The finite set theory is equivalent (in some sense) to arithmetic (Kaye and Wong [29]); consistency of these theories is nearly indubitable,[20] in contrast to the (full) set theory whose axiom of infinity says basically that the class is a set.[21]

In the finite set theory, the class of numbers may be defined as the class of all sets such that is transitive, that is, and is totally ordered by membership, that is, Adding the condition that is non-empty, that is, we get the class of natural numbers

Each set is equinumerous to one and only one ; as before, "equinumerous" means existence of a set such that that is, where ), and is a one-to-one correspondence between and that is, In this case we say that is the number of members of

The sum of may be defined as the number of members in the disjoint union The product of may be defined as the number of members in the set product The power for may be defined as the number of functions from to (that is, from to ).

A rational number could be defined as an equivalence class of triples of natural numbers w.r.t. such equivalence relation: when (informally this means that of course). However, in this case we cannot introduce the class of rational numbers (since a proper class cannot be member of a class). Thus, it is better to choose a single element in each equivalence class, and define a rational number as a triple of numbers such that at least one of the two numbers is and the other is coprime to (or ). We get the class of all rational numbers. And, in order to treat natural numbers as a special case of rational numbers, we identify each natural number with the corresponding rational number [22]

Back to definability. We want to endow the class (of all sets in the finite set theory) with the D-structure generated by the membership relation True, the notion of a D-structure on transcends the finite set theory, since a collection of classes is neither a set nor a class. But still, in the metatheory, a class may be called definable when it is obtainable from the membership relation by the 5 operations (complement, union, permutation, Cartesian product, projection) introduced in Section 2 for relations on the real line in particular, and arbitrary set in general. However, a pair of real numbers is not a real number, while a pair of sets is a set! That is, and are disjoint; in contrast, The order relation "" between real numbers is a subset of (rather than ). But what about the membership relation "" between sets ? Should we treat it as a subclass of or ?

True, the plane is not a subset of the line but it can be injected into by a definable function; recall the injection introduced in the end of Section 3 and used in Section 6 for encoding binary relations by unary relations. For example, the binary relation is encoded by the unary relation

Here is a general lemma basically applicable in both situations, and (though, in the latter case it needs some adaptation to the proper class).

Lemma.   Let be a set endowed with a D-structure, and a definable injection. Then a binary relation is definable if and only if the unary relation is definable.

  • Exercise 9.2. Prove this lemma.  Hint: "If":   "Only if":
  • Exercise 9.3. All relations on mentioned above are definable classes. Prove it.   Hint: the equality relation "" is ""; the relation "" is ""; the ternary relation "" is ""; the ternary relation "" is ""; the lemma applies; further, is the intersection of the class of transitive sets and the class of sets totally ordered by membership, etc. etc., up to "".

Similarly, the basic relations between rational numbers are definable classes.

Real numbers cannot be represented by finite sets, but can be represented by classes (of finite sets) in several ways. In the spirit of Dedekind cuts we treat a real number as the class of all rational numbers smaller than this real number. More formally: a real number is a subclass of such that

  • is a lower class; that is,
  • contains no greatest element; that is
  • is not empty, and not the whole that is, and

And, in order to treat rational numbers as a special case of real numbers, we identify each rational number with the corresponding real number

Some examples. The real number ("the Pythagoras' constant") is the class of all rational numbers such that The golden mean is the class of all rational numbers such that The real number is the class of all rational numbers such that

Can we define via factorials, as in Exercise 5.13? We can define factorials without recursion; is the number of bijective functions from to itself (that is, from to itself; in other words, permutations). But still, we need recursion when defining partial sums of the series for Generally, an infinite sequence of rational numbers is the class of pairs Specifically, the sequence of partial sums of is the class of pairs such that (and and of course). But we cannot define a class by its property! We deal with a D-structure on A class must be defined by a common property of all its members, not a property of the class. Otherwise it would be second-order definability in (thus, a transfinite level of the cumulative hierarchy). Can we formulate the appropriate property of a pair alone? Yes, we can. Here is the property: there exists a function such that and The clue is that a finite segment of the infinite sequence (of partial sums) is enough.

Similarly, an infinite sequence of sets is the class of pairs and it can be defined recursively, by a recurrence relation of the form where is a definable class of pairs, and an initial condition for (Use finite segments of the infinite sequence.)

Thus, every computable sequence of natural (or rational) numbers is a definable class of pairs. No need to use Diophantine sets. Rather, for every Turing machine, all possible "complete configurations" (called also "situations" and "instantaneous descriptions") may be treated as elements of a subclass of and the rule of transition from one complete configuration to the next complete configuration may be treated as a definable class of pairs (of complete configurations).

It follows that every computable real number, and moreover, every limit computable real number is definable. Having a convergent definable sequence of rational numbers, we define its limit as the class of rational numbers such that In particular, (the Archimedes' constant) and (the Chaitin's constant) are definable.

A sequence of real numbers cannot be treated as the class of pairs (since is not a set), but can be treated as the disjoint union that is, the set of pairs where (recall a similar workaround in Section 6). Also, a continuous function cannot be treated as the class of pairs but can be treated as the class of pairs of rational numbers such that Such precautions allow us to translate basic calculus into the language of finite set theory. However, arbitrary functions and arbitrary subsets of are unavailable. Thus, the continuum hypothesis makes no sense. Also, transferring measure theory and related topics (especially, theory of random processes) to this ground (as far as possible) requires effort and ingenuity.

The finite set theory can provide a reliable alternative airfield for much (maybe most) of the mathematical results especially important for applications, in case of catastrophic developments in the transfinite hierarchy. Several possible such "alternative airfields" are examined by mathematicians and philosophers [30], [31], [32], [33], [34], [35], [36].

Informally, the finite set theory uses (for infinite classes) the idea of potential infinity, prevalent before Georg Cantor, while the transfinite hierarchy uses the idea of actual (completed) infinity, prevalent after Georg Cantor.[23]

Transfinite hierarchy

The transfinite part of the cumulative hierarchy begins with the first transfinite ordinal number (an infinite set) and the first transfinite stage of the hierarchy (an infinite set; ). Note that implies but implies rather We continue as before:

and so on; we get the stages for all finite and again, The union of all these stages is the stage (still an infinite set), and (an infinite subset of ). Again, implies Let us dwell here before climbing higher.

Encoding of various mathematical objects by sets is somewhat arbitrary (see Wikipedia:Equivalent definitions of mathematical structures; likewise, an image may be encoded by files of type jpeg, gif, png etc.), and their places in the hierarchy vary accordingly. Treating a pair as and a triple as we get (for )

Treating the set of natural numbers as we get Treating a rational number as an equivalence class of triples of natural numbers we get where is the set of all rational numbers. Alternatively, treating an integer as an equivalence class of pairs of natural numbers, and a rational number as an equivalence class of pairs of integers,[24] we get

here is the set of all integers. Treating a real number as a set of rational numbers we get

where is the set of all real numbers, and is such that be it or anyway, it follows that

Taking into account that generally and (since ), we get and for all Every subset of belongs to and every set of subsets of belongs to in particular, the σ-algebra of all Lebesgue measurable subsets of belongs to Also, every function belongs to and every set of such functions belongs to in particular, every equivalence class (under the relation of equality almost everywhere) of Lebesgue measurable functions belongs to and the set of all equivalence classes of Lebesgue integrable functions belongs to And the set of all bounded linear operators belongs to Clearly, a lot of notable mathematical objects belong to [25]

Would something like suffice for all the objects mentioned above? The answer is negative as long as is defined as where means For every the relation is ensured by the two exercises below.

  • Exercise 9.4. If then Prove it.  Hint:
  • Exercise 9.5. If for some then Prove it.  Hint: induction in and the previous exercise.

A more economical encoding is available (and was used in Section 6, see Exercises 6.1, 6.2); instead of the set of all -tuples we may use the set of all -sequences ; as before, is the set of pairs.

  • Exercise 9.6. If then for all Prove it.  Hint:

A lot of theorems are published about real numbers, real-valued functions of real arguments, spaces of such functions etc. I wonder, is there at least one such theorem sensitive to the distinction between and ? That is, theorem that can be formulated and proved within but not ? I guess, the answer is negative. A seemingly similar question: is definability of real numbers sensitive to the distinction between and ? I mean, is there at least one real number definable in but not ? This time, the answer is affirmative, as explained below.

For each we endow the set with the D-structure generated by the membership relation (for of course).

Recall that, treating a real number as a set of rational numbers, and a rational number as an equivalence class of triples of natural numbers, we have and That is, and

Similarly to the finite set theory, and are definable subsets of (whenever ), and the basic relations between natural numbers are definable, as well as the basic relations between rational numbers. Dissimilarly to the finite set theory, is a definable subset of (whenever ), and the basic relations between real numbers are definable. An example: for we have Another example: for we have Also the relation "" between a rational number and the corresponding real number is definable, which implies definability of embedded into Thus, all real numbers first-order definable in (as in Section 3) are definable in (whenever ).

What about second-order definability? It was treated in Section 5 as a D-structure on the set but a more economically encoded set may be used equally well.

  • Exercise 9.7. Prove it.  Hint: use Exercise 9.6.

Moreover, for every is a definable subset of ; and the four relations (that generate the D-structure in Section 5) are definable relations on Thus, all real numbers second-order definable as in Section 5 are definable in whenever In particular, the "first-order undefinable but second-order definable" number of Section 6 is definable in However, all said does not mean that it is undefinable in

What we need is the second-order definability in rather than ; that is, definability in the set

  • Exercise 9.8. Prove it.  Hint: similar to Exercise 9.7.

Once again, is a definable subset of and all real numbers definable in are definable in That is, all real numbers second-order definable in are (first-order) definable in

A straightforward generalization of Section 6 gives a real number second-order definable in but first-order undefinable in This number is definable in but undefinable in Similarly, for each there exist real numbers definable in but undefinable in We observe an infinite hierarchy of definability orders within

Climbing higher on the cumulative hierarchy we get stages for ordinal numbers such as Still higher, then then Everyone may continue until feeling too dizzy; see Wikipedia:Ordinal notation, Ordinal collapsing function, Large countable ordinal. All these are countable ordinals. By the way, every countable ordinal may be visualized by a set of rational numbers, using a strictly increasing function (that is, ). For example, may be visualized by

For every countable ordinal there exist real numbers definable in but undefinable in Moreover, some of these real numbers are of the form (recall Section 7), since there exists an increasing sequence (of natural numbers) definable in that overtakes all sequences definable in

A wonder: stages for like are as far from ordinary mathematics as numbers like from ordinary engineering. Nevertheless these contribute to the supply of definable real numbers.

Still higher, the set of all countable ordinals is the first uncountable ordinal It cannot be visualized by a set of rational or real numbers. Its cardinality is the first uncountable cardinality The continuum hypothesis is equivalent to the equality between and the cardinality continuum.

For every ordinal (countable or not) the set of all real numbers definable in is countable (and moreover, has an enumeration definable in ). In particular, the set of all real numbers definable in is countable. On the other hand, new definable real numbers emerge on all countable levels, and there are uncountably many such levels. A contradiction?!

No, this is not a contradiction. Denoting by the set of all real numbers definable in and by the set of all ordinals definable in we have wherever (which follows from the lemma of Section 5). For all countable ordinals mentioned before we have (that is, all ordinals below are definable in ). In contrast, since is countable. The union contains all real numbers definable with ordinal parameters ; but definability with parameters is outside the scope of this essay (recall Section 2).

It is natural to ask, whether for all countable ordinals or not. Probably, we only know that the affirmative answer cannot be proved without the choice axiom, and do not know, which answer (if any) can be proved with the choice axiom.[26]

If for all countable ordinals then and is uncountable (of the cardinality of ). Otherwise, there exists a countable ordinal such that while for all (and therefore for some ). In this case the transfinite sequence is not monotone, and we do not know, whether the union is countable, or not.

Countability or uncountability of matters for model dependence. There is a countable set of formulas (in the language of the set theory) that define real numbers on all levels Some are model independent, others are model dependent. If is countable, then each of these model dependent definable real numbers has at most countably many possible values. Otherwise, if is uncountable, then at least one of these model dependent definable real numbers has uncountably many possible values.

This matter is closely related to the position of Laureano Luna [37] (see also [38, pages 19-20]):

"Pieces of language taken as mere syntactical expressions (letter-strings) should be distinguished from definitions, which are semantical objects, namely, interpreted letter-strings." (Page 61.)
"The meanings of the letter-strings that express definitions of reals are context-dependent, the context being here the definability level on which they are used. [...] if some letter-strings express more than one definition of a real number, there is no reason to think there are only countably many such definitions and only countably many definable real numbers." (Page 64.)

Two objections arise. First, we did not prove that is uncountable. Second, model dependence does not apply to since is not a model of ZFC. We'll return to the second objection after climbing on the cumulative hierarchy to and much higher.

The stage of the cumulative hierarchy is vast; its cardinality is very large (much larger than the cardinality of ). Now consider the first ordinal of this very large cardinality, and the corresponding stage Iterating this jump we get a slight idea of the class of all sets, the incredible universe of the set theory ZFC. The whole grows from a small seed, the first infinite ordinal whose existence is just postulated (the axiom of infinity).

If you want to soar above you need a new axiom of infinity that ensures existence of an ordinal such that is a model of ZFC; every such ordinal, being initial, is a cardinal, called a worldly cardinal. For climbing still higher try the so-called large cardinals. And be assured that these supernal stages do contribute to the supply of definable real numbers.[27]

Assuming existence of large cardinals we get a transfinite hierarchy of worldly cardinals and the corresponding models of ZFC, for all countable ordinals (and more; but here we do not need uncountable ). Using instead of we get new versions of and Again, we do not know, whether this new is countable, or not. If it is uncountable, then again, at least one model dependent definable real number has uncountably many possible values; and this time, model dependence applies.

Getting rid of undefinable numbers

Climbing down to earth, is it possible to restrict ourselves to definable numbers and still use the existing theory of real numbers and related objects? An affirmative answer was found in 1952 [40] and enhanced recently [39].

Before climbing down we need to climb up to the first worldly cardinal and the corresponding model of ZFC. Within this model we consider the constructible hierarchy take the least such that is a model of ZFC, and get the so-called minimal transitive model of ZFC. This model is pointwise definable [1, "Minimal transitive model"], that is, every member of this model is definable (in this model).

Accordingly, this model is countable (and is countable). Nevertheless, every theorem of ZFC holds in every model of ZFC; in particular, Cantor's theorem " is uncountable" holds in the countable model No contradiction; enumerations of exist, but do not belong to Likewise, a well-known theorem of measure theory states that the interval cannot be covered by a sequence of intervals of total length True, for every the set being countable, can be covered by a sequence of intervals of total length but such sequences do not belong to (even if endpoints do belong). Working in we have to ensure that all relevant objects (not only real numbers) belong to

  • One often hears it said that since there are indenumerably many sets and only denumerably many names, therefore there must be nameless sets. The above shows this argument to be fallacious. (Myhill 1952, see [40, the last paragraph].)
  • In my opinion, an object is conceivable only if it can be defined with a finite number of words. (Poincare 1910, translated from German, see [37, page 58].)

Conclusion

Each definition (of a real number, or another mathematical object) is a finite text in a language. The language may be formal (mathematical) or informal (natural). In both cases the text is composed of expressions that refer to objects and relations between objects. The extension of an expression is the corresponding set of objects, or set of pairs (of objects), or triples, and so on. For a mathematical language, all objects are mathematical; a natural language may mention non-mathematical objects, and even itself, as in the phrase "The preceding two paragraphs are an expression in English that unambiguously defines a real number " (recall Introduction, Richard's paradox), which leads to a problem: the mentioned expression in English fails to define! "So when we speak in English about English, the ‘English’ in the metalanguage is not exactly the same as the ‘English’ in the object-language." [38, p. 15]. A natural language, intended to be its own metalanguage, is burdened with paradoxes. A mathematical language avoids such (and hopefully, any) paradoxes at the expense of being different from its metalanguage. The metalanguage is able to enumerate all real numbers definable in the language and define more real numbers.

On one hand, a natural language itself is inappropriate for mathematics. On the other hand, a mother tongue is always a natural language. In order to avoid both restrictions of a fixed mathematical theory and paradoxes of a natural language we may get the best of both worlds by considering two-part texts. The first part, written in a natural language, introduces a mathematical theory. The second part, written in the (mathematical) language of this theory, defines some real number.

In this framework the question "is every real number definable?" falls out of mathematics, because the notion "mathematical theory" above cannot be formalized. Some may admit only potential infinity and stop on the finite set theory. Some may admit actual infinity and the transfinite hierarchy up to (exclusively) some preferred ordinal (sometimes [42]; more often, something controversially believed to be the first undefinable ordinal[28]). Or the whole universe of ZFC but no more (equivalently, up to the first worldly cardinal).[29] Or the Tarski-Grothendieck set theory. Or higher, up to some preferred large cardinal. Or some more exotic alternative set theory. Or something brand new, like a kind of homotopy type theory. Or even something not yet published. Most choices mentioned above were unthinkable in the first half of the 20th century. Who knows what may happen near year 2100? "Mathematics has no generally accepted definition" (from Wikipedia:Definitions of mathematics); the same can be said about "mathematical theory".[30] Admitting actual infinity we do not get rid of potential infinity; the latter returns, hardened, on a higher level [34], [41]. Maybe the recent, hotly debated conception of multiverse is another fathomable segment of the unfathomable potential infinity of mathematics. "The most general definition of a definition" appears to be as problematic as "the set of all sets".

Another problem manifests itself as model dependence for a mathematical language, and context dependence for a natural language. In both cases a single text may refer to different objects (depending on the model or the context, respectively), which blurs the idea of definability. Recall Section 8 (the last paragraph): every real number is "hardwired" in some model of the set theory (ZFC). We may feel that definability of the number does not follow unless the model is definable; but what do we mean by definability of a model? Another case, recall Section 9.2 (the last paragraph): uncountable hierarchy of models indexed by countable ordinals leads to a set of real numbers, possibly uncountable (but maybe not). Nothing is "hardwired" here, except for these countable ordinals. Outside mathematics, a more or less similar case, treated by Luna [37], shows context dependence in a hierarchy of contexts (levels of definability) indexed by definable countable ordinals. A mathematical counterpart with uncountably many definable real numbers could exist in some alternative (to ZFC and alike; probably closer to Kripke–Platek) set theory such that the class of all definable countable ordinals is not a countable set, and preferably, not a set at all [37, p. 65].

Bad news: definability is a very subtle property of a real number. Good news: other properties, more relevant to applications, are unsubtle; and definability is rather of philosophical interest. "Mathematicians, in general, do not like to deal with the notion of definability; their attitude toward this notion is one of distrust and reserve." (Tarski [42], the first phrase; now partially obsolete, partially actual.)

Competing interests

The author has no competing interest.

Notes and references

  1. Whichever definition of 'definable' you choose, the formula that defines a definable object is a finite sequence of characters belonging to a finite alphabet. Thus the set of definable objects is definitively countable. (From "Wikipedia". A talk page. 2018.)
  2. Classical mathematics permits (and requires) the existence of undefinable objects, but many people find this philosophically disquietening, questioning how an object can be said to exist if no mathematical statement can be used to uniquely identify it.
    As a result, a few mathematicians have developed systems of mathematics that do not involve undefinable objects. (From "Wikipedia". Obsolete version of an article. 2004.)
  3. The describable numbers are all numbers for which there is any possible finite description that uniquely identifies the number. The countability argument still works: you can still enumerate all possible finite-length strings that could be descriptions, and define a one-to-one correspondence between strings that could be descriptions and natural numbers. The set of describable numbers is, thus, still countable, and the set of undescribables is not, which implies that the set of undescribables is far, far larger that the describables. (From: Chu-Carroll, Mark C. (2014). "You can't even describe most numbers!". Good Math/Bad math blog. {{cite web}}: External link in |last1= (help))
  4. Tsirelson, Boris (2003). "Reminiscences". Self-published.
  5. Because depends on the program encoding used, it is sometimes called Chaitin's construction instead of Chaitin's constant when not referring to any specific encoding. (From Wikipedia:Chaitin's Constant.) The polynomial used in this definition is not uniquely determined as "such that the sequence is uncomputable", but for now every such polynomial serves our purpose; later, in Section 8, we'll add one more requirement related to the uncomputability. Other requirements, related to randomness, are irrelevant to this essay. Existence of such follows from Matiyasevich's theorem; 9 unknowns are sufficient according to: Jones, James (1982). "Universal Diophantine equation". J. Symbolic Logic (Cambridge) 47 (3): 549-571. doi:10.2307/2273588. https://doi.org/10.2307/2273588. 
  6. See Wikipedia:Von Neumann–Bernays–Gödel set theory and Gödel operation.
  7. Aaronson, Scott (1999). "Who can name the bigger number?" (PDF). Self-published.
  8. Baez, John (2012). "Enormous integers". Self-published.
  9. "Are there any sizes in between?" This question (in the case of a negative answer, it is the continuum hypothesis), is one of the most famous in set theory. Much as in the case of the parallel postulate, it was widely believed that the continuum hypothesis could simply be proven from ZFC, and Cantor and many others devoted enormous time and effort to developing such a proof. It was not until much later that the combined efforts of Gṏdel and Cohen established once and for all:
    Theorem 3 (Gṏdel, Cohen) The continuum hypothesis is independent of ZFC.   (From: J. Reitz [12, Section 3].)
  10. Therefore one may on good reason suspect that the role of the continuum problem in set theory will be this, that it will finally lead to the discovery of new axioms which will make it possible to disprove Cantor's conjecture.   (From: K. Gṏdel [14, the end].)
    It was Gödel who first suggested that perhaps "strong axioms of infinity" (large cardinals) could decide interesting set-theoretical statements independent over ZFC, such as CH. This hope proved largely unfounded for CH — one can show that virtually all large cardinals defined so far do not affect the status of CH.   (From: R. Honzik [13, Abstract].)
  11. A point of view which the author feels may eventually come to be accepted is that CH is obviously false. [...] This point of view regards as an incredibly rich set given to us by one bold new axiom, which can never be approached by any piecemeal process of construction. Perhaps later generations will see the problem more clearly and express themselves more eloquently.   (From: P. Cohen [15, p. 151].)
    If we really believe that the set-theoretic universe has to be built up piecemeal we surely cannot accept an axiom according to which enormous new sets (enormous because there is a jump in cardinality) simply nonconstructively appear.   (From: Nik Weaver [16, p. 5].)
  12. Many set theorists yearn for a definitive solution of the continuum problem, what I call a dream solution, one by which we settle the continuum hypothesis (CH) on the basis of a new fundamental principle of set theory, a missing axiom, widely regarded as true, which determines the truth value of CH. [...] If achieved, a dream solution to the continuum problem would be remarkable, a cause for celebration.
    In this article, however, I shall argue that a dream solution of CH has become impossible to achieve. Specifically, what I claim is that our extensive experience in the set-theoretic worlds in which CH is true and others in which CH is false prevents us from looking upon any statement settling CH as being obviously true.   (From: J. Hamkins [17].)
    Such a situation with the continuum problem raises doubts about the widely used set theory, especially the axiom of powerset. Alternative, more constructive approaches attract the attention of mathematicians and philosophers [18], [19], [20], [21]. Maybe the collection of all subsets of an infinite set should be treated as a class rather than a set.
  13. 2.3 mln articles in science and engineering are published in 2014, of them 2.6%=0.06 mln in mathematics. (See "National Center for Science and Engineering Statistics". 2018.) Also, 0.3 mln articles were submitted to arXiv till now, of them 0.03 mln in 2014. (See "arXiv submission rate statistics". 2017.) Thus, probably, 0.6 mln math articles are published for now. I guess, the average number of theorems per article is at least 2, which gives 1.2 mln published theorems. Some of them are notable, many are so-so, some are not new. And probably, hundreds or even thousands of them only pretend to be theorems, because of unnoticed errors in proofs. On the other hand, numerous lemmas formally are theorems. True, authors usually build proofs in the framework of the relevant branch of mathematics; but nearly all branches are embedded into ZFC.
    Today ZFC is the standard form of axiomatic set theory and as such is the most common foundation of mathematics. (From Wikipedia:ZFC.)
  14. Most notable exception, not the only exception. About 30 exceptions are available in Wikipedia:List of statements independent of ZFC.
  15. Wikipedia, "Law of excluded middle"; also "Gelfond-Schneider constant".
  16. From Encyclopedia of Mathematics:Creative set.
  17. In set theory, we have the phenomenon of the universal definition. This is a property first-order expressible in the language of set theory, that necessarily holds of exactly one set, but which can in principle define any particular desired set that you like, if one should simply interpret the definition in the right set-theoretic universe. So could be defining the set of real numbers or the integers or the number or a certain group or a certain topological space or whatever set you would want it to be. For any mathematical object there is a set-theoretic universe in which is the unique object for which
    Theorem. Any particular real number can become definable in a forcing extension of the universe.
    (From: J. Hamkins "The universal definition". Self-published. 2017. See also [24].)
    We adapt this idea. For notation used here, see Wikipedia:Ordinal number, in particular, Section "Ordinals and cardinals"; and by the way, Also, following Kunen [25], the relation between two initial ordinals and is interpreted as the same relation between the corresponding cardinals. Thus, is the initial ordinal of the cardinality of the continuum.
    For every sequence of binary digits the set is a Easton index function [25, Section 8.4, Def. 4.1]; Easton forcing [25, Section 8.4, Th. 4.7; also Corollary 4.8] gives a model of ZFC such that for all and (and the model has the same cardinals as the metatheory). Note that in this model (since and (since
    We consider the second-order definable set of all disjoint unions of sets such that is equinumerous to and for each
    • is equinumerous to some subset of
    • is not equinumerous to
    • every subset of not equinumerous to is equinumerous to some subset of
    (It means that is of cardinality We note that equinumerosity of and is third-order definable. If is empty, we let Otherwise, for every we define to be if and are equinumerous for some (therefore, all) sets otherwise (It means that Finally, we choose a definable map from onto and let
  18. So ecumenical set theorists instead spin this amazing structure from only the set that does not depend on the existence of anything: the empty set. This is the closest mathematicians get to creation from nothing! (From "Nothingness". Stanford Encyclopedia of philosophy. 2017.)
  19. From the standpoint of mainstream mathematics, the great foundational debates of the early twentieth century were decisively settled in favor of Cantorian set theory, as formalized in the system ZFC (Zermelo-Fraenkel set theory including the axiom of choice). Although basic foundational questions have never entirely disappeared, it seems fair to say that they have retreated to the periphery of mathematical practice. Sporadic alternative proposals like topos theory or Errett Bishop’s constructivism have never attracted a substantial mainstream following, and Cantor’s universe is generally acknowledged as the arena in which modern mathematics takes place.   (From: Nik Weaver [16, the first paragraph].)
  20. The vast majority of contemporary mathematicians believe that Peano's axioms are consistent, relying either on intuition or the acceptance of a consistency proof such as Gentzen's proof. A small number of philosophers and mathematicians, some of whom also advocate ultrafinitism, reject Peano's axioms because accepting the axioms amounts to accepting the infinite collection of natural numbers. (From Wikipedia:Peano axioms.)
  21. "There is something profoundly unsatisfactory about the axiom of infinity. It cannot be described as a truth of logic in any reasonable use of this term and so the introduction of it as a primitive proposition amounts in effect to the abandonment of Frege's project of exhibiting arithmetic as a development of logic" (Kneale and Kneale, p. 699).
    This patched up set theory could not be identified with logic in the philosophical sense of "rules for correct reasoning." You can build mathematics out of this reformed set theory, but it no longer passes as a foundation, in the sense of justifying the indubitability of mathematics. (From [26, p. 148–149].)
  22. In many cases of interest there is a standard (or "canonical") embedding, like those of the natural numbers in the integers, the integers in the rational numbers, the rational numbers in the real numbers, and the real numbers in the complex numbers. In such cases it is common to identify the domain with its image contained in so that (From Wikipedia:Embedding.)
  23. In the philosophy of mathematics, the abstraction of actual infinity involves the acceptance (if the axiom of infinity is included) of infinite entities, such as the set of all natural numbers or an infinite sequence of rational numbers, as given, actual, completed objects. This is contrasted with potential infinity, in which a non-terminating process (such as "add 1 to the previous number") produces a sequence with no last element, and each individual result is finite and is achieved in a finite number of steps. (From Wikipedia:Actual infinity.)
    Before Cantor, the notion of infinity was often taken as a useful abstraction which helped mathematicians reason about the finite world; for example the use of infinite limit cases in calculus. The infinite was deemed to have at most a potential existence, rather than an actual existence. (From Wikipedia:Controversy over Cantor's theory.)
    The difficulty with finitism is to develop foundations of mathematics using finitist assumptions, that incorporates what everyone would reasonably regard as mathematics (for example, that includes real analysis). (Ibid., Objection to the axiom of infinity.)
  24. In Wikipedia see Integer:Construction, and Rational number:Formal construction.
  25. is the universe of "ordinary mathematics", and is a model of Zermelo set theory. (From Wikipedia:Von Neumann universe.)
  26. In ZF (Zermelo-Frenkel set theory without the choice axiom) we can do the following.
    For arbitrary countable ordinal the set of all -definable (that is, definable in ) relations on is countable, and has a -definable enumeration. Doing so for all simultaneously we get a -definable function such that Restricting ourselves to of the form (where ) we get a -definable function such that
    Now, for every sequence of countable ordinals, the union is not just a countable union of countable sets, but a countable union of sets enumerated simultaneously by some function (which is trivial in ZFC but nontrivial in ZF), and therefore their union is countable, hence, not the whole On the other hand, existence of such that is consistent with ZF (Feferman and Lévy 1963, see Cohen [15, p. 143]). Thus, it is consistent with ZF that
    This matter is closely related to a question about another definable (in or otherwise) family, see MathStackExchange "Is it possible to define a family of fundamental sequences for all countable (limit) ordinals? (Without AC)"., especially, the answer by Noah Schweber, and Remark 24 in Section 2.1 of Forster, Thomas E. "A tutorial on countable ordinals" (PDF). Self-published.
    Another question of this kind, "Is the smallest with undefinable ordinals always countable?". is answered affirmatively by Miha Habič.
  27. Eventually, we will in our definitions be attracted to the possibilities of using higher order mathematical objects and constructions, such as function classes, spaces or measures, and this amounts to defining objects in increasingly large fragments of the set-theoretic universe. Most all of the classical mathematical structure is itself definable in the set-theoretic structure a model of the Zermelo axioms, and so the definable reals of this structure includes almost every real ever defined classically. The structures arising with larger ordinals, however, allow us to define even more reals. (From [39, Section 1].)
  28. Thus, our definability levels cannot go beyond the countable definable ordinals. What these are is contentious. Constructivists will typically argue that all ordinals are constructive and that the (classical) least nonconstructive ordinal does not exist, though from more liberal standpoints it is a definable countable ordinal. Some predicativists believe that the classical Feferman-Schütte ordinal does not exist (though from the classical viewpoint it is a definable countable ordinal); some, as Weaver 2009, believe predicativity can go beyond (Luna [37, p. 64].)
  29. Someone may say: I just use ZFC "as is" and do not care about worldly cardinals. But in this case many interesting definable real numbers are model dependent (recall Section 8), they exploit large cardinals whenever possible.
  30. The set of all consistent effectively axiomatized formal theories is well-defined but irrelevant, because most of them have no intended interpretation.

Bibliography

[1] Hamkins, Joel David (2012). Must there be numbers we cannot describe or define? (PDF). Set theory seminar at the University of Bristol.

[2] Doyle, Peter G. (2011). "Maybe there's no such thing as a random sequence". arXiv:1103.3494 (self-published).

[3] Finch, Steven R. (2003). Mathematical constants. Cambridge University. doi:10.1017/CBO9780511550447. ISBN 0-521-81805-2. 

[4] Poonen, Bjorn (2008). "Undecidability in number theory". Notices AMS (Amer. Math. Soc.) 55 (3): 344-350. http://www-math.mit.edu/~poonen/papers/h10_notices.pdf. 

[5] van den Dries, Lou (1998). O-minimal structures and real analytic geometry (PDF). Current Developments in Mathematics 1998. Boston: International Press (published 1999). pp. 105–152. doi:10.4310/CDM.1998.v1998.n1.a4. ISBN 978-1-571-46077-6.

[6] van den Dries, Lou (1998). "Introduction and overview". Tame Topology and O-minimal Structures. Cambridge University. doi:10.1017/CBO9780511525919.004. ISBN 978-0-521-59838-5. http://inis.jinr.ru/sl/vol1/UH/_Ready/Mathematics/van%20den%20Dries%20L.%20Tame%20topology%20and%20O-minimal%20structures%20(Cambridge,%201998)(L)(96s).pdf. 

[7] Booij, Auke Bart (2013). "The structure of tame and wild sets". Bachelor thesis.

[8] Macintyre, Angus (2016). Structures and first-order definable relations (PDF). AGTA2016. Slides for mini-course.

[9] Tyrrell, Brian (2017). "An analysis of tame topology using O-minimality" (PDF). Bachelor's thesis.

[10] Basu, Saugata; Pollack, Richard; Roy, Marie-Françoise (2006). Algorithms in real algebraic geometry. Springer. doi:10.1007/3-540-33099-2. ISBN 978-3-540-33098-1. http://perso.univ-rennes1.fr/marie-francoise.roy/bpr-ed2-posted1.html. 

[11] Marker, David; Messmer, Margit; Pillay, Anand (2017). Model theory of fields (2nd ed.). Cambridge University Press. ISBN 978-1-107-16807-7. https://books.google.co.il/books?id=m9scDgAAQBAJ. 

[12] Reitz, Jonas (2017). "From geometry to geology: an invitation to mathematical pluralism through the phenomenon of independence". Journal of Indian Council of Philosophical Research (Springer India) 34 (2): 289-308. doi:10.1007/s40961-017-0099-5. https://doi.org/10.1007/s40961-017-0099-5. 

[13] Honzik, Radek (2018). "Large cardinals and the continuum hypothesis". In Antos C.. The Hyperuniverse Project and Maximality. Birkhäuser, Cham. pp. 205-226. doi:10.1007/978-3-319-62935-3_10. ISBN 978-3-319-62934-6. https://link.springer.com/chapter/10.1007/978-3-319-62935-3_10. 

[14] Gödel, Kurt (1947). "What is Cantor's continuum problem?". The American Mathematical Monthly 54 (9): 515-525. doi:10.2307/2304666. http://www.jstor.org/stable/2304666. 

[15] Cohen, Paul (1966). Set theory and the continuum hypothesis. W. A. Benjamin. ISBN 978-0-805-32327-6. https://books.google.co.il/books?id=ZKc-AAAAIAAJ. 

[16] Weaver, Nik (2005). "Mathematical conceptualism". arXiv:math:0509246 (self-published).

[17] Hamkins, Joel David (2015). "Is the dream solution of the continuum hypothesis attainable?". Notre Dame Journal of Formal Logic 56 (1): 135-145. doi:10.1215/00294527-2835047. https://projecteuclid.org/euclid.ndjfl/1427202977.  Also arXiv:1203.4026.

[18] Rathjen, Michael (2016). "Indefiniteness in semi-intuitionistic set theories: on a conjecture of Feferman". The Journal of Symbolic Logic 81 (2): 742-753. doi:10.1017/jsl.2015.55. https://doi.org/10.1017/jsl.2015.55.  Also arXiv:1405.4481.

[19] Lingamneni, Shivaram (2020). "Can we resolve the continuum hypothesis?". Synthese (Springer Verlag) 197: 599–622. doi:10.1007/s11229-017-1648-9. https://doi.org/10.1007/s11229-017-1648-9. 

[20] Koellner, Peter (2017). "Feferman on set theory: infinity up on trial". In Gerhard Jäger. Feferman on Foundations. Outstanding Contributions to Logic, vol 13.. Springer, Cham. pp. 491-523. doi:10.1007/978-3-319-63334-3_19. ISBN 978-3-319-63332-9. https://link.springer.com/chapter/10.1007%2F978-3-319-63334-3_19. 

[21] Parsons, Charles (2017). "Feferman's skepticism about set theory". In Gerhard Jäger. Feferman on Foundations. Outstanding Contributions to Logic, vol 13.. Springer, Cham. pp. 525-543. doi:10.1007/978-3-319-63334-3_20. ISBN 978-3-319-63332-9. https://link.springer.com/chapter/10.1007%2F978-3-319-63334-3_20. 

[22] Solovay, Robert M. (2000). "A version of Ω for which ZFC cannot predict a single bit". In C.S. Calude. Finite Versus Infinite. Discrete Mathematics and Theoretical Computer Science. Springer, London. pp. 323-334. doi:10.1007/978-1-4471-0751-4_21. ISBN 978-1-85233-251-8. https://doi.org/10.1007/978-1-4471-0751-4_21. 

[23] Calude, Cristian S. (2002). "Chaitin Ω numbers, Solovay machines, and Gödel incompleteness". Theoretical Computer Science (Elsevier) 284 (2): 269-287. doi:10.1016/S0304-3975(01)00068-8. https://doi.org/10.1016/S0304-3975(01)00068-8. 

[24] Hamkins, Joel David; Woodin, W. Hugh (2017). "The universal finite set". arXiv:1711.07952 (self-published).

[25] Kunen, Kenneth (1980). Set theory. An introduction to independence proofs. Elsevier. https://books.google.co.il/books?id=U09djgEACAAJ. 

[26] Hersh, Reuben (1997). What is mathematics, really?. Oxford University. ISBN 9780195130874. http://books.google.com/books?isbn=0195130871. 

[27] Takahashi, Moto-o (1977). "A foundation of finite mathematics". Publications of the Research Institute for Mathematical Sciences (Kyoto Univ.) 12 (3): 577-708. doi:10.2977/prims/1195190375. https://doi.org/10.2977/prims/1195190375. 

[28] Baratella, Stefano; Ferro, Ruggero (1993). "A theory of sets with the negation of the axiom of infinity". Mathematical Logic Quarterly 39 (1): 338-352. doi:10.1002/malq.19930390138. https://doi.org/10.1002/malq.19930390138. 

[29] Kaye, Richard; Wong, Tin Lok (2007). "On interpretations of arithmetic and set theory". Notre Dame Journal of Formal Logic 48 (4): 497-510. doi:10.1305/ndjfl/1193667707. https://doi.org/10.1305/ndjfl/1193667707. 

[30] Feferman, Solomon (1992). "Why a little bit goes a long way: logical foundations of scientifically applicable mathematics". PSA: Proceedings of the Biennial Meeting of the Philosophy of Science Association (University of Chicago) (2): 442-455. doi:10.1086/psaprocbienmeetp.1992.2.192856. https://doi.org/10.1086/psaprocbienmeetp.1992.2.192856. 

[31] Feferman, Solomon (2013). "How a little bit goes a long way: predicative foundations of analysis" (PDF). (self-published).

[32] Weaver, Nik (2005). "Analysis in ". arXiv:math:0509245 (self-published).

[33] Simpson, Stephen G. (2009). Subsystems of second order arithmetic (2nd ed.). Cambridge University. doi:10.1017/CBO9780511581007. ISBN 978-0-511-58100-7. https://doi.org/10.1017/CBO9780511581007. 

[34] Simpson, Stephen G. (2014). "An objective justification for actual infinity?". In Chitat Chong. Infinity and Truth. World Scientific. pp. 225-228. doi:10.1142/9789814571043_0008. ISBN 978-981-4571-03-6. https://doi.org/10.1142/9789814571043_0008. 

[35] Weaver, Nik (2009). "Predicativity beyond ". arXiv:math:0509244 (self-published).

[36] Storer, Timothy James (2010). "A defence of predicativism as a philosophy of mathematics". Dissertation (Cambridge).

[37] Luna, Laureano (2017). "Rescuing Poincaré from Richard’s paradox". History and Philosophy of Logic (Taylor & Francis) 38 (1): 57-71. doi:10.1080/01445340.2016.1247322. https://doi.org/10.1080/01445340.2016.1247322. 

[38] Luna, Laureano; Taylor, William (2010). "Cantor's proof in the full definable universe". The Australasian Journal of Logic 9: 10-26. doi:10.26686/ajl.v9i0.1818. https://doi.org/10.26686/ajl.v9i0.1818. 

[39] Hamkins, Joel David; Linetsky, David; Reitz, Jonas (2013). "Pointwise definable models of set theory". The Journal of Symbolic Logic 78 (1): 139-156. doi:10.2178/jsl.7801090. https://doi.org/10.2178/jsl.7801090.  Also arXiv:1105.4597.

[40] Myhill, John (1952). "The hypothesis that all classes are nameable". Proceedings of the National Academy of Sciences 38 (11): 979-981. doi:10.1073/pnas.38.11.979. https://doi.org/10.1073/pnas.38.11.979. 

[41] Linnebo, Øystein (2013). "The potential hierarchy of sets". The Review of Symbolic Logic 6 (2): 205-228. doi:10.1017/S1755020313000014. https://doi.org/10.1017/S1755020313000014. 

[42] Tarski, Alfred (1956). "On definable sets of real numbers". Logic, Semantics, Metamathematics: Papers from 1923 to 1938. Oxford, Clarendon Press. pp. 110-142. ISBN 978-0-91514-475-4. https://books.google.co.il/books?id=2uhra9PEFZsC. 

Index

Cantor's diagonal argument

cardinality continuum
CH, the continuum hypothesis
Chaitin's constant
computability
computable
  — number
  — sequence
continuum hypothesis
countable
cumulative hierarchy
D-structure (generated by)
definable
  — function
  — number
  — relation
  — sequence
digit
disjoint union
equinumerous
infinity
  — actual
  — potential
injection
Matiyasevich theorem
model dependent
model independent
number
  — chosen at random
  — computable
  — definable
  — limit computable
  — uncomputable
  — undefinable
operation
  — Boolean (union, complement)
  — permutation
  — projection
  — set multiplication
overtake
paradox, Richard's
pointwise definable
predicate
  — binary
  — ternary
  — unary
product of sets, Cartesian
relation
  — addition
  — appendment
  — binary
  — definable
  — membership
  — multiplication
  — -ary
  — order
  — second-order
  — ternary
  — unary
  — undefinable
second-order definable
semialgebraic
set
  — Diophantine
  — recursively enumerable
strictly increasing
transposition, adjacent
tuple
  — versus finite sequence
undefinable
  — number
  — relation
worldly cardinal
ZFC, axiomatic set theory

the -th digit
evaluation
the set of all natural numbers
power set
set of relations
the set of all real numbers
set of all tuples and relations
injects -dim to 1-dim
injects 2-dim to 1-dim
the set of all integers

the first transfinite ordinal